Đến nội dung

Hoang72 nội dung

Có 536 mục bởi Hoang72 (Tìm giới hạn từ 05-05-2020)



Sắp theo                Sắp xếp  

#736638 VMF's Marathon Đa thức Olympic

Đã gửi bởi Hoang72 on 05-01-2023 - 18:48 trong Đa thức

Bài 9 đã 1 tuần chưa có lời giải, mình xin được đăng tiếp 1 bài để tiếp tục topic:

Bài 10: Hỏi có tồn tại hay không đa thức $f(x)$ hữu tỉ, có bậc bằng $4$ và thỏa mãn:

$1: f(x) \geq \sqrt{2}; \forall x \in \mathbb{R}.$

$2: \exists x_0 \in \mathbb{R}: f(x_0)= \sqrt{2}.$

Giả sử tồn tại đa thức $f(x) thoả mãn yêu cầu bài toán.

Nhận thấy $x_0$ là điểm cực tiểu của $f(x)$ nên $f'(x_0) = 0$.

Ta chứng minh đa thức tối tiểu của $x_0$ trên $\mathbb Q[x]$ có bậc $3$.

Thật vậy, giả sử phản chứng, thì ta có $x_0 = a+b\sqrt{c}$, với $a,b,c\in\mathbb Q$ và 

$\Rightarrow f(a + b\sqrt{c}) = A + B\sqrt{c}$, với $A,B\in\mathbb Q$.

Do đó $A+B\sqrt{c} = \sqrt{2}\Rightarrow A = 0; B^2 . c = 2$.

Thế thì $f(a - b\sqrt{c}) = A - B\sqrt{c} = -\sqrt{2} < \sqrt{2}$, mâu thuẫn với giả thiết.

Do đó đa thức tối tiểu của $x_0$ trên $\mathbb Q[x]$ có bậc ít nhất là $3$, mà $f'(x_0) = 0$ nên đa thức tối tiểu của $x_0$ là $f'(x)$. Chú ý rằng ở đây ta không quan tâm hệ số cao nhất của đa thức tối tiểu.

Đến đây có $2$ hướng giải:

Hướng 1

Hướng 2 (của thầy Lê Phúc Lữ)




#736609 $f^2(x)+f^2(y) \leq 2f(xy), \forall x,y \in \mathbb...

Đã gửi bởi Hoang72 on 02-01-2023 - 20:42 trong Phương trình hàm

Thay $x=y$ ta có $f^2(x)\leq f(x^2),\forall x\in\mathbb R$.

Do đó $f(x)\geq 0,\forall x\in\mathbb R$.

Thay $y=1$ ta có $\left[f(x) - 1\right]^2 \leq 1 - f^2(1),\forall x\in\mathbb R$. $(*)$

Suy ra $f$ bị chặn. 

Giả sử tồn tại $x_0$ sao cho $|f(x_0)|> 1$.

Bằng quy nạp, dễ dàng chứng minh: $f\left(x_0^{2^n}\right) \geq f^{2^n}(x_0),\forall n\in\mathbb N^*$.

Suy ra $\lim_{n\to +\infty} f\left(x_0^{2^n}\right) = +\infty$, vô lí vì $f$ bị chặn.

Do đó với mọi $x\in\mathbb R$ thì $f(x)\in \{0; 1; -1\}$

$\Rightarrow f(1)\in \{0; 1\}$ (Do $f(1)\geq 0$).

Nếu $f(1) = 1$ thì từ $(*)$ ta có $f(x) = 1,\forall x\in\mathbb R$.

Nếu $f(1) = 0$ thì thay $y = \frac{1}{x}$ ta có $f^2(x) + f^2\left(\frac{1}{x}\right)\leq 0\Rightarrow f(x) = 0,\forall x\in\mathbb R$.

Vậy...




#736606 $xf\left(x+y\right)\ge\left(y+1\right)f\le...

Đã gửi bởi Hoang72 on 02-01-2023 - 20:16 trong Phương trình hàm

Kí hiệu $P(a;b)$ là phép thế $x=a;y=b$ vào BPTH đã cho.

Nhận xét 1: $\begin{cases} f(x) \geq 1,\forall x>1 \\ f(x)\leq 1,\forall x < 1\end{cases}$:

Chứng minh

Nhận xét 2: $0\leq f(1)\leq 1$

Chứng minh

Nhận xét 3: $f(1)= 1$

Chứng minh

Bây giờ, ta thấy $f(1) =1$ nên $f(x)\geq x,\forall x\in\mathbb R$.

Ở phương trình hàm ban đầu, cho $x\leq0$ ta có: $x(x+y)\geq (y+1)f(x)+x-1,\forall x\leq0, y\in\mathbb R$

$\Rightarrow \frac{x^2+xy-x+1}{y+1}\geq f(x),\forall x\leq0, y > -1$.

Cho $y\to+\infty$ ta có ngay $f(x)\leq x,\forall x \leq 0\Rightarrow f(x) = x,\forall x \leq 0$.

$P(-1; y+1)\Rightarrow -f(y)\geq -(y+2)-2,\forall y\in\mathbb R\Rightarrow f(y)\leq y+4,\forall y\in\mathbb R$.

Do đó ở phương trình hàm ban đầu ta có: $x(x+y+4)\geq (y+1)f(x)+x-1,\forall x,y>0$

$\Rightarrow f(x)\leq \frac{x^2+xy+3x+1}{y+1},\forall x,y>0$.

Cho $y\to+\infty$ ta có $f(x)\leq x,\forall x>0$.

Như vậy $f(x) = x,\forall x\in\mathbb R$. Thử lại ta thấy thoả mãn.

Vậy ta có hàm duy nhất thoả mãn $f(x)=x,\forall x\in\mathbb R$.




#736584 $f(x+y+f(y))=f(f(x))+2y$

Đã gửi bởi Hoang72 on 01-01-2023 - 00:03 trong Phương trình hàm

Kí hiệu $P(a;b)$ là phép thế $x=a;y=b$ vào phương trình hàm đã cho.

$P(x;0)\Rightarrow f(x+f(0)) = f(f(x)),\forall x\in\mathbb R$. $(1)$

$P(0;y)\Rightarrow f(y+f(y)) = f(f(0)) + 2y,\forall y\in\mathbb R$. $(2)$

Ta chứng minh hàm số $y+f(y)$ là toàn ánh.

Thật vậy, ta có: $f(f(x)) + f(x) = f(x + f(0)) + f(x),\forall x\in\mathbb R$.

Do đó để chứng minh $y+f(y)$ là toàn ánh, ta đi chứng minh hàm số $f(x+f(0)) + f(x)$ là toàn ánh.

Tuy nhiên, từ đề bài ta có:

\begin{align*} f(x+y + f(y)) + f(f(0) + x + y + f(y)) &= f(f(x)) + 2y + f(f(x + f(0))) + 2y \\&= f(f(x)) + f(f(x+f(0))) + 4y\end{align*}

Cố định $x$, ta suy ra $f(x) + f(x+f(0))$ là hàm số toàn ánh.

Dẫn đến $y+f(y)$ là hàm số toàn ánh.

Bây giờ, kết hợp $(1),(2)$ và thay vào phương trình hàm đã cho ta có: $f(x + y+f(y)) = f(x + f(0)) + f(y+f(y)) - f(f(0)),\forall x,y\in\mathbb R$

Mà $y+f(y)$ là toàn ánh nên $f(x+y) = f(x+f(0)) + f(y) - f(f(0)),\forall x,y\in\mathbb R$

$\Rightarrow f(x) + f(y) = f(x+y-f(0)) + f(f(0)),\forall x,y\in\mathbb R$.

Do đó từ $(2)$ ta có: $f(f(y)) + f(y + f(0))  - 2f(f(0))=2y,\forall y\in\mathbb R$.

Kết hợp với $(1)$, ta có $f(y+f(0)) = y+f(f(0)),\forall y\in\mathbb R$

$\Rightarrow f(y) = y + f(f(0)) - f(0),\forall y\in\mathbb R$. (thoả mãn)

Vậy $f(x) = x+c,\forall x\in\mathbb R$.

 




#736583 Chứng minh rằng $n=2^k, k \geq 1$ hoặc $n=3.2^k, k \...

Đã gửi bởi Hoang72 on 31-12-2022 - 23:38 trong Số học

Với mọi $m\in\mathbb N^*\setminus \{1\}$, đặt $A_m = \{(i,j)\in \{1;2;...;m-1\}^2,i\neq j: m\mid 2i+j\}$.

Gọi $P(n)$ là mệnh đề thoả mãn $P(n)$ đúng khi tồn tại các số nguyên $x_1,x_2,...,x_{n-1}$ thoả mãn yêu cầu thoả mãn, ngược lại $P(n)$ sai.

Trước tiên, ta chứng minh nếu $P(n)$ sai, với $n,p\in\mathbb N^*\setminus \{1\}$ thì $P(np)$ sai.

Thật vậy, giả sử $P(np)$ đúng. Ta thấy $A_n . p  \subset A_{np}$, trong đó kí hiệu $A_n . p$ là tập gồm tích các phần tử của $A_n$ với $p$.

Do $A_{np}$ đúng nên tồn tại các số $x_{p}, x_{2p},...,x_{(n-1)p}$ thoả mãn với mọi $0<i,j<n; i\neq j$ và $np\mid 2ip+jp$ thì $x_{ip} < x_{jp}$.

Đặt $y_i = x_{pi},\forall i = \overline{1,n-1}$. Thế thì $y_1,y_2,...,y_{n-1}$ thoả mãn với mọi $(i,j)\in A_n$, ta có $y_i < y_j$.

Từ đây ta suy ra $P(n)$ đúng, vô lí.

Do đó để chứng minh bài toán đã cho, ta chỉ cần chứng minh trường hợp $n$ là số nguyên tố lẻ lớn hơn $3$ thì không tồn tại các số nguyên $x_1,x_2,..,x_{n-1}$ thoả mãn.

Ta phát biểu lại mệnh đề trên như sau:

Cho đồ thị có hướng $G=(V;E)$ có $n$ đỉnh, trong đó $n$ là số nguyên tố lẻ lớn hơn $3$. Các đỉnh được đánh số từ $1$ đến $n$. Với mọi $i\neq j$, có một cạnh nối từ đỉnh $i$ đến đỉnh $j$ khi và chỉ khi $2i + j\in \{n; 2n\}$. Khi đó nếu đồ thị không có chu trình thì $n=2^k,k\in\mathbb N^*$ hoặc $n=3.2^k,k\in\mathbb N$.

Chứng minh:

Từ đề bài ta có các cạnh của đồ thị là: $\begin{cases}1\to 2u-1\\2 \to 2u-3 \\ ... \\ u\to 1 \end{cases}$ và $\begin{cases} u+1\to 2u \\ u+2\to 2u-2 \\ ... \\ 2u \to 2\end{cases}$.

Nhìn vào bảng trên, ta thấy mỗi đỉnh đều là xuất phát của đúng một cạnh và kết thúc của đúng một cạnh. Đồng thời, không có hai đỉnh nào có tính chất tự phản (Đỉnh này đi đến đỉnh kia và ngược lại).

Thế thì ta xét một đường đi bắt đầu từ đỉnh $1$: Đỉnh $1$ đến được duy nhất một đỉnh, đỉnh tiếp theo cũng đi đến được duy nhất một đỉnh... Nếu trong đường đi có gặp lại một đỉnh nào đó, ta được một chu trình. Ngược lại, đường đi này sẽ lặp lại vô hạn bước, và ta có điều vô lí.

Do đó đồ thị luôn có ít nhất một chu trình. Ta có đpcm.




#736581 Chứng minh tồn tại số nguyên dương $M$ sao cho: $a_{m...

Đã gửi bởi Hoang72 on 31-12-2022 - 21:46 trong Số học

Cám ơn bạn nhé làm sao để nghĩ ra lời giải như vầy nhỉ

Thường những bài như này hay dùng định giá p - adic nên mình nghĩ đến chứng minh $v_p(a_{n+1})\leq v_p(a_n)$. Trong quá trình làm mới phát hiện ra $v_p(a_n)-v_p(a_1)$ luôn cùng dấu nên có lời giải như vậy.




#736577 Chứng minh tồn tại số nguyên dương $M$ sao cho: $a_{m...

Đã gửi bởi Hoang72 on 31-12-2022 - 17:33 trong Số học

Đặt $s_n = \frac{a_1}{a_2} + \frac{a_2}{a_3}+...+\frac{a_{n-1}}{a_n}+\frac{a_n}{a_1}$.

Ta có $s_n\in\mathbb Z,\forall n\geq N$ nên $s_{n+1} - s_n\in\mathbb Z,\forall n\geq N$

$\Rightarrow \frac{a_n}{a_{n+1}} + \frac{a_{n+1}}{a_1} - \frac{a_n}{a_1}\in\mathbb Z,\forall n\geq N$.

Lấy $p$ là số nguyên tố bất kì. 

Nhận xét: Với mọi $n\geq N$: Nếu $v_p(a_n)> v_p(a_1)$ thì $v_p(a_{n+1})> v_p(a_1)$. Ngược lại, nếu $v_p(a_n)\leq v_p(a_1)$ thì $v_p(a_{n+1})\leq v_p(a_1)$

Chứng minh: Giả sử phản chứng.

TH1: $v_p(a_n)> v_p(a_1)$: Khi đó theo giả sử thì $v_p(a_{n+1}) \leq v_p(a_1) < v_p(a_n)$

$\Rightarrow v_p\left(\frac{a_{n+1}}{a_1}\right) < v_p\left(\frac{a_n}{a_1}\right) \leq v_p\left(\frac{a_n}{a_{n+1}}\right)$

$\Rightarrow v_p\left(\frac{a_n}{a_{n+1}} + \frac{a_{n+1}}{a_1} - \frac{a_n}{a_1}\right) = v_p\left(\frac{a_{n+1}}{a_1}\right) < 0$, vô lí vì $\frac{a_n}{a_{n+1}} + \frac{a_{n+1}}{a_1} - \frac{a_n}{a_1}\in\mathbb Z$.

TH2: $v_p(a_n) \leq v_p(a_1)$: Khi đó theo giả sử thì $v_p(a_n) \leq v_p(a_1) < v_p(a_{n+1})$

$\Rightarrow v_p\left(\frac{a_n}{a_{n+1}}\right) < v_p\left(\frac{a_n}{a_1}\right) <v_p\left(\frac{a_{n+1}}{a_1}\right)$

$\Rightarrow v_p\left(\frac{a_n}{a_{n+1}} + \frac{a_{n+1}}{a_1} - \frac{a_n}{a_1}\right) = v_p\left(\frac{a_n}{a_{n+1}}\right) < 0$, vô lí vì $\frac{a_n}{a_{n+1}} + \frac{a_{n+1}}{a_1} - \frac{a_n}{a_1}\in\mathbb Z$.

Từ nhận xét trên, ta thấy $v_p(a_n) > v_p(a_1),\forall n\geq N$ hoặc $v_p(a_n) \leq v_p(a_1),\forall n \geq N$.

 

$\bullet$ Nếu $v_p(a_n) > v_p(a_1),\forall n\geq N$ thì $v_p(a_n)\geq v_p(a_{n+1}),\forall n\geq N$.

Thật vậy, do $v_p\left(\frac{a_{n+1}}{a_1} - \frac{a_n}{a_1}\right) >0,\forall n\geq N$ và $v_p\left(\frac{a_n}{a_{n+1}} + \frac{a_{n+1}}{a_1} - \frac{a_n}{a_1}\right),\forall n\geq N$ nên $v_p\left(\frac{a_n}{a_{n+1}}\right)\geq 0,\forall n\geq N$

$\Rightarrow v_p(a_n)\geq v_p(a_{n+1}),\forall n\geq N$.

Mà $\left \{ v_p(a_n)\right \}_{n=1}^{+\infty}$ là dãy số nguyên không âm nên đến lúc nào đó nó là hằng số.

 

$\bullet$ Nếu $v_p(a_n)\leq v_p(a_1),\forall n\geq N$ thì $v_p(a_n)\leq v_p(a_{n+1}),\forall n\geq N$.

Thật vậy, giả sử tồn tại $n\geq N$ sao cho $v_p\left(a_{n+1}\right) < v_p\left(a_{n}\right)$.

Thế thì $v_p\left(\frac{a_{n+1}}{a_1}\right) < v_p\left(\frac{a_n}{a_1}\right) \leq v_p\left(\frac{a_n}{a_{n+1}}\right)\Rightarrow v_p\left(\frac{a_n}{a_{n+1}} + \frac{a_{n+1}}{a_1} - \frac{a_n}{a_1}\right) = v_p\left(\frac{a_{n+1}}{a_1}\right) < 0$, vô lí

$\Rightarrow v_p(a_{n+1})\geq v_p(a_n),\forall n\geq N$.

Tương tự, do dãy $\left \{ v_p(a_n)\right \}_{n=1}^{+\infty}$ bị chặn trên nên đến một lúc nào đó nó là hằng số.

 

Bây giờ, ta chứng minh dãy $(a_n)$ chỉ chứa hữu hạn ước nguyên tố. Thật vậy, từ hai trường hợp trên ta thấy $p\mid a_{n+1}$ khi và chỉ khi $p\mid a_na_1$.

Do đó bằng quy nạp, ta dễ dàng chỉ ra số ước nguyên tố của dãy là hữu hạn.

Đồng thời, với mọi số nguyên tố $p$ thì dãy $\left \{v_p(a_n)\right\}$ là hằng số kể từ thời điểm nào đó nên suy ra $(a_n)$ là hằng số kể từ chỉ số nào đó. 

P/s: Lúc đầu mình có một số nhầm lẫn nên giờ mình sửa lại.




#736576 cho tam giác ABC nhọn nội tiếp (O), Xét lục giác lồi MNPQRS ,.. CMR MN+PQ+RS...

Đã gửi bởi Hoang72 on 31-12-2022 - 16:00 trong Hình học phẳng

P636.png
Lấy $X$ đối xứng với $B$ qua $OM$; $Y$ đối xứng với $B$ qua $OS$.
Khi đó dễ thấy $OX = OB = OY$ nên $X,Y\in (O)$.
Đồng thời, ta có $\widehat{XOY} = \widehat{XOB} + \widehat{YOB}=2\widehat{MOB}+2\widehat{SOB} = 2\widehat{SOM}=2\widehat{ABC}=\widehat{AOC}$
$\Rightarrow XY = AC$.
Do đó: $BS+SM+MB = YS + SM + MX \geq XY = AC$
$\Rightarrow BS + SM + BM\geq CA$.
Tương tự: $\begin{cases} CN+CP+NP \geq AB \\ AR+AQ+RQ \geq BC \end{cases}$.
Cộng vế với vế ba bất đẳng thức trên lại ta được:
$BS+SM+BM+CN+CP+NP+AR+AQ+RQ\geq CA+AB+BC$
$\Rightarrow SM + NP + RQ\geq (CA-CP - AQ) + (AB-AR - BS) + (BC - BM - CN)$
$\Rightarrow SM+NP+RQ\geq PQ+RS+MN$.
Ta có điều phải chứng minh.



#736566 Tìm số nguyên dương $m$ nhỏ nhất sao cho: $2f_{1}(m)...

Đã gửi bởi Hoang72 on 30-12-2022 - 18:33 trong Số học

Nhận thấy nếu $4\mid m$ thì $f_1(m) = f_1\left(\frac{m}{2}\right); f_2(m) = f_2\left(\frac{m}{2}\right)$.

Do đó ta chỉ xét khi $4\nmid m$. Vì $f_2(m)>0$ nên $m$ chẵn.

Đặt $m=2p_1^{k_1}...p_s^{k_s} . p_{s+1}^{k_{s+1}}...p_{t}^{k_t}$, trong đó $p_1,p_2,...,p_s$ là các số nguyên tố dạng $4k+1$ và $p_{s+1},p_{s+2},...,p_t$ là các số nguyên tố dạng $4k+3$.

Thế thì $f_2(m)$ cũng chính là số ước của $\frac{m}{2}$ và bằng $\prod_{i=1}^t (k_t+1)$.

Ta tính $f_1(m)$. Lấy $d = p_1^{h_1}p_2^{h_2}...p_s^{h_s}.p_{s+1}^{h_{s+1}}...p_t^{h_t}$ là ước nguyên tố của $n$ thì $h_i\leq k_i,\forall i = \overline{1,t}$ và $2\mid h_{s+1}+...+h_t$.

Có $(k_1+1)...(k_s+1)$ cách chọn bộ $(h_1,h_2,...,h_s)$. Ta xét hai trường hợp:

$\bullet$ TH1: Trong các số $k_{s+1},...,k_t$ có ít nhất một số lẻ: Không mất tính tổng quát giả sử $k_t$ lẻ.

Chọn bộ $(h_{s+1},...,h_{t-1})$ có $(k_{s+1}+1)...(k_{t-1} + 1)$ cách chọn. Với mỗi bộ như vậy có đúng $\frac{k_t+1}{2}$ cách chọn $h_t$ để $h_t$ và $h_{s+1}+...+h_{t-1}$ cùng tính chẵn lẻ.

Do đó số cách chọn bộ $(h_{s+1},...,h_{t})$ là $\frac{(k_{s+1}+1)...(k_{t} + 1)}{2}$.

$\bullet$ TH2: $2\mid k_{s+1},...,k_t$: 

Nếu $h_t <k_t$ thì theo TH1 số bộ thoả mãn là $\frac{(k_{s+1}+1)...(k_{t-1}+1)k_t}{2}$.

Nếu $h_t = k_t$ và $h_{t-1} < k_{t-1}$ thì theo TH1 số bộ thoả mãn là $\frac{(k_{s+1}+1)...(k_{t-2}+1)k_{t-1}}{2}$

...

Nếu $h_t = k_t; h_{t-1}=k_{t-1};...;h_{s+2}=k_{s+2}$ và $h_{s+1} < k_{s+1}$ thì số bộ thoả mãn là $\frac{k_{s+1}}{2}$.

Nếu $h_i = k_i,\forall i = \overline{s+1,t}$ thì số bộ thoả mãn là $1$.

Bằng khai triển dễ dàng chứng minh được: $\frac{(k_{s+1}+1)...(k_{t} + 1)+1}{2} = \frac{(k_{s+1}+1)...(k_{t-1}+1)k_t}{2} + \frac{(k_{s+1}+1)...(k_{t-2}+1)k_{t-1}}{2}+...+\frac{k_{s+1}}{2} + 1$

Do đó số bộ thoả mãn là $\frac{(k_{s+1}+1)...(k_{t} + 1)+1}{2}$

$\Rightarrow f_1(m) = (k_1+1)...(k_s+1)\left \lfloor \frac{(k_{s+1}+1)...(k_{t} + 1)+1}{2}\right \rfloor$.

Từ đẳng thức $2f_1(m) - f_2(m) = 2017$ ta phải có $(k_1+1)...(k_s+1) = 2017$

$\Leftrightarrow s = 1; k_1 = 2016$.

Vậy $m_{\min} = 2.5^{2016}$.

 




#736561 Chứng minh có thể chọn ra một vài số trong $n$ số này mà có tổng bằ...

Đã gửi bởi Hoang72 on 30-12-2022 - 15:03 trong Tổ hợp và rời rạc

Đặt $m = 2019$.

Do $m(m-1)(m-2)\mid S$ nên $S\geq m(m-1)(m-2)$.

Nếu trong các số $a_1,a_2,...,a_n$, mỗi số trong tập $\{1;2;...;m\}$ chứa nhiều nhất $m-2$ số thì $$2S \leq (m-2)(1 + 2 + ... + m)< 2m(m-1)(m-2)\text{ (vô lí)}$$

Do đó tồn tại $i\in \{1;2;...;m\}$ sao cho $i$ xuất hiện ít nhất $m-1$ lần.

Đặt $A$ là đa tập chứa $m-1$ số $i$ này và $B$ là đa tập chứa các số còn lại. Ta có $|A| = m-1$ và $|B| = n - (m-1)$.

Nếu $n - (m-1) \leq i$ thì $a_1+a_2+...+a_n \leq mi + (m-1)i = (2m-1)i<2S$, vô lí.

Suy ra $n-(m-1) > i$. Do đó ta chọn được một lượng số không quá $i$ phần tử từ $B$ có tổng chia hết cho $i$.

Cho các phần tử này vào đa tập mới là $C$ và xoá các phần tử này trong $B$. 

Do $\sum_{x\in B}x = 2S - (m-1)i$ nên ta làm được thao tác trên liên tục cho đến khi tổng các phần tử của $T$ lớn hơn $S - mi$ thì dừng lại.

Thật vậy, nếu $\sum_{x\in T} x \leq S - mi$ thì khi đó tổng các phần tử của $B$ ít nhất là $S + i$, tức $|B| \geq i$, vô lí.

Đồng thời nếu $\sum_{x\in T} x > S$ thì trước đó ta đã thêm vào $T$ một số số có tổng lớn hơn $mi$, tức ta đã thêm vào ít nhất $i+1$ số, vô lí vì mỗi lần ta thêm không quá $i$ số.

Do đó ta điều chỉnh được sao cho $S - mi < \sum_{x\in T} x \leq S$.

Vì $i\mid S$ và $i\mid \sum_{x\in T} x$, ta có thể đặt $\sum_{x\in T}x = S - ki$, với $k\in \{0;1;...;m-1\}$.

Bổ sung $k$ phần tử $i$ của $A$ vào $T$, ta được một tổng bằng $S$. (đpcm)




#736560 Chứng minh $m \leq 159$.

Đã gửi bởi Hoang72 on 30-12-2022 - 14:04 trong Tổ hợp và rời rạc

Gọi $a_1,a_2,...,a_{66}$ lần lượt là số đường thẳng đi qua $66$ điểm này.

Thế thì $m = \sum_{i=1}^{66} a_i$.

Ta đếm số bộ $(M,\{b,c\})$ trong đó $M\in A; b,c\in B; \{A\} = b \subset c$.

Đếm theo $M$, ta thấy số bộ như vậy là $\sum_{i=1}^{66} \binom{a_i}{2}$.

Đếm theo $b,c$, ta thấy số bộ như vậy không vượt quá $\binom{16}{2}$.

Do đó $\sum_{i=1}^{66} \binom{a_i}{2}\leq \binom{16}{2}\Leftrightarrow \sum_{i=1}^{66} \left(a_i^2 - a_i\right) \leq 240$.

Với mọi $x\in\mathbb Z$ thì $(x-2)(x-3)\geq 0\Rightarrow x^2\geq 5x-6$.

Dẫn đến $4\sum_{i=1}^{66} a_i- 6.66 \leq 240\Leftrightarrow \sum_{i=1}^{66}a_i\leq 159$.

Vậy $m\leq 159$.




#736418 $\lim (\sum x_i)/n=0$ thì $\lim (\sum f(x_...

Đã gửi bởi Hoang72 on 23-12-2022 - 11:15 trong Phương trình hàm

$\bullet$ Chứng minh $f$ là hàm lẻ:

$\bullet$ Chứng minh $f$ là hàm cộng tính:

Nhờ tính chất cộng tính, điều kiện bài toán tương đương: Với mọi dãy $(y_n)$ tiến về $0$ thì $\lim \frac{f(ny_n)}{n} = 0$.

Mặt khác $f(ny_n) = nf(y_n)$ nên $\lim f(y_n) =0$.

Suy ra $f$ liên tục tại $0$.

Do $f$ liên tục tại $0$ và $f$ cộng tính nên $f(x) = ax,\forall x\in\mathbb R$, với $a$ là hằng số nào đó.

 




#736404 $f(m+n) + f(m-n) = 2f(m)f(n)$

Đã gửi bởi Hoang72 on 22-12-2022 - 17:15 trong Phương trình hàm

Kí hiệu $P(a;b)$ là phép thế $x=a;y=b$ vào phương trình hàm đã cho.

$P(0;0)\Rightarrow 2f(0) = 2f^2(0)\Rightarrow f(0) \in \{0; 1\}$.

$\bullet$ $f(0) = 0$: $P(m;m)\Rightarrow f(2m) = 2f^2(m),\forall m\in\mathbb Z$. $(*)$

Giả sử tồn tại $a\in\mathbb Z\setminus \{0\}$ sao cho $f(a) \neq 0$.

Từ $(*)$, bằng quy nạp ta có $f(2^ka) = \frac{(2f(a))^{2^k}}{2},\forall k\in\mathbb N^*$.

Mặt khác $f$ bị chặn nên điều trên là vô lí khi cho $k\to+\infty$.

Do đó $f(m) = 0,\forall m\in\mathbb Z$.

$\bullet$ $f(1) = 1$: $P(m;m)\Rightarrow f(2m) = 2f^2(m)  - 1,\forall m\in\mathbb Z$. $(**)$

Giả sử tồn tại $b$ sao cho $|f(b)| \geq 2$.

Tồn tại $r\in\mathbb R; |r|>1$ sao cho $f(b) = \frac{1}{2}\left(r+\frac{1}{r}\right)$.

Từ $(**)$ quy nạp ta được $f(2^kb) =  \frac{1}{2}\left(r^{2^k} + \frac{1}{r^{2^k}}\right),\forall k\in\mathbb N^*$.

Cho $k\to+\infty$ ta thấy vô lí. Do đó $f(m)\in \{0; \pm 1\},\forall m\in\mathbb Z$.

Từ $(**)$ dễ dàng chứng minh được $f(m) = 1,\forall 4\mid m$.

Do đó cho $m=2(2k+1), n = 2(2h+1),k,h\in\mathbb Z$ vào PTH ban đầu ta có: $2f(2(2k+1))f(2(2h+1)) = 2\Rightarrow f(2(2k+1))f(2(2h+1)) = 1$

$\Rightarrow f(2(2k+1)) = f(2(2h+1)) = \pm 1,\forall k,h\in\mathbb Z$.

Đặt $\begin{cases} A_1 = \{x\in\mathbb Z : 4\mid x\} \\ A_2 = \{x\in\mathbb Z : x\equiv 2\pmod 4 \} \\ A_3 = \{x\in\mathbb Z: 2\nmid x\}\end{cases} $. Thế thì $A_1,A_2,A_3$ là ba phân hoạch của $\mathbb Z$.

Ta có $f(x) = 1,\forall x\in A_1$, và $f(x) =1,\forall x\in A_2$ hoặc $f(x) = -1,\forall x\in A_2$.

Nếu $f(x) = 1,\forall x\in\mathbb A_2$ thì $f(x) = 1,\forall 2\mid x$.

Ở PTH ban đầu cho $m,n$ lẻ thì $f(m)f(n) = 1\Rightarrow f(m)  = 1,\forall m\in A_3$ hoặc $f(m) = -1,\forall m \in A_3$.

Thử lại ta có hai hàm thoả mãn là $f(m) = 1,\forall m\in\mathbb Z$ hoặc $f(x) = \begin{cases} 1 \text{ nếu m chẵn} \\ -1 \text{ nếu m lẻ}\end{cases}$.

Nếu $f(x) = -1,\forall x\in A_2$ thì cho $m=n$ và $m$ lẻ ta có $f(m) = 0,\forall m\in A_3$.

Thử lại ta thấy thoả mãn.

Vậy tổng cộng có ba hàm thoả mãn:

$f(m) = 1,\forall m\in\mathbb Z$;

$f(m) = \begin{cases} 1 \text{ nếu m chẵn} \\ -1 \text{ nếu m lẻ}\end{cases}$;

$f(m) = \begin{cases} 1 \text{ nếu 4}\mid m \\ -1\text{ nếu } m\equiv 2\pmod 4 \\ 0\text{ nếu 2}\nmid m\end{cases}$

 




#736402 Cho a>0. Tìm $f: \mathbb{R}^{^{+}...

Đã gửi bởi Hoang72 on 22-12-2022 - 16:16 trong Phương trình hàm

Giả sử tồn tại hàm số thoả mãn yêu cầu bài toán.

Đặt $b=\frac{a}{2} > 0$. Phương trình hàm đã cho tương đương\begin{equation} f\left(\frac{1}{x} + f(y) \right) = \frac{1}{f(x)} + by,\forall x,y>0 \end{equation}

Thay $y$ bởi $\frac{1}{z} + f(y)$ vào $(1)$ ta có: $f\left(\frac{1}{x} + f\left(\frac{1}{z} + f(y)\right)\right) = \frac{1}{f(x)} + \frac{b}{z} + bf(y),\forall x,y,z>0$

$\Leftrightarrow f\left(\frac{1}{x} + \frac{1}{f(z)} + by\right) = \frac{1}{f(x)} + \frac{b}{z} + bf(y),\forall x,y,z>0$.

Ở trên, thay $x$ bởi $\frac{1}{bx}$ ta có $f\left(bx + \frac{1}{f(z)} + by\right)=\frac{1}{f\left(\frac{1}{bx}\right)} + \frac{b}{z}+bf(y),\forall x,y,z>0$.

Hoán vị $x,y$ ta có $\frac{1}{f\left(\frac{1}{bx}\right)} +bf(y) = \frac{1}{f\left(\frac{1}{by}\right)} + bf(x),\forall x,y>0\Leftrightarrow bf(x) - \frac{1}{f\left(\frac{1}{bx}\right)} = c,\forall x > 0$, với $c$ là hằng số nào đó.

+ Nếu $c<0$ thì $\frac{1}{f\left(\frac{1}{bx}\right)} = bf(x) - c > -c,\forall x>0$

$\Rightarrow \frac{1}{f(x)}> -c,\forall x>0\Rightarrow f(x) < -c,\forall x>0$.

Tuy nhiên ở $(1)$ cho $y\to+\infty$ ta thấy ngay điều vô lí.

+ Nếu $c>0$ thì $bf(x)>c,\forall x>0\Rightarrow f(x) > \frac{c}{b} ,\forall x > 0$

$\Rightarrow bf(x) = \frac{1}{f\left(\frac{1}{bx}\right)} + c < \frac{b}{c} + c,\forall x > 0$, hay $f$ bị chặn trên.

Tương tự trường hợp trên ta thấy điều vô lí.

Do đó $c=0$, hay $f\left(\frac{1}{bx}\right) = \frac{1}{bf(x)},\forall x > 0$ $(2)$

Thay $x$ bởi $\frac{1}{bx}$ vào $(1)$ rồi sử dụng $(2)$ ta có $f\left(bx + f(y)\right) = bf(x) + by,\forall x,y>0$ $(3)$

Dễ thấy $f$ là đơn ánh. 

Ở $(3)$ thay $y$ bởi $f(y)$ ta có $f(bx + f(f(y))) = bf(x) + bf(y),\forall x,y>0$. $(4)$

Hoán vị $x,y$ thì $f(bx + f(f(y))) = f(by + f(f(x))),\forall x,y>0$

$\Rightarrow bx + f(f(y)) = by + f(f(x)),\forall x,y>0\Rightarrow f(f(x)) = bx + d,\forall x > 0$, với $d$ là hằng số không âm.

Ở trên, thay $x$ bởi $\frac{1}{bx}$ ta có: $f\left(f\left(\frac{1}{bx}\right)\right) = \frac{1}{x} + c,\forall x > 0$.

Theo $(2)$ thì $f\left(\left(\frac{1}{bx}\right)\right) = f\left(\frac{1}{bf(x)}\right) = \frac{1}{bf(f(x))} = \frac{1}{b(bx+c)},\forall x > 0$.

Do đó $\frac{1}{b(bx+c)} = \frac{1}{x}+c,\forall x >0$.

Để điều này đúng với mọi $x>0$ thì $b=1$ và $c=0$.

Suy ra $f(f(x)) = x,\forall x > 0$, và từ $(4)$ ta có $f(x+y) = f(x) + f(y),\forall x,y>0$.

Dẫn đến $f$ là hàm tăng và là hàm cộng tính, nên $f(x) =mx,\forall x > 0$. Thử lại ta có $m=1$.

Vậy để phương trình hàm đã cho có nghiệm thì $a=2$ và khi đó phương trình hàm có nghiệm duy nhất $\boxed{f(x) = x,\forall x >0}$.

 




#736367 $\sum\limits_{} {{{\left( {...

Đã gửi bởi Hoang72 on 20-12-2022 - 14:40 trong Số học

Với mọi $i\in \{0; 1;...; 2^{n-1} - 1\}$ ta có: $\displaystyle\binom{2^n}{2i+1}= \frac{2^n(2^n-1)...(2^n - 2i)}{1.2...(2i+1)}$.

Do $i < 2^{n-1}$ nên $v_2(2^n - 2k) = v_2(2k),\forall k = \overline{1,i}$

$\begin{aligned} \Rightarrow v_2((2^n-1)(2^n-2)...(2^n-2i)) &= v_2(2^n - 2)+v_2(2^n-4)+...+v_2(2^n-2i)\\&=v_2(2) + v_2(4) + ... + v_2(2i) \\&= v_2(1.2...(2i+1))\end{aligned}$.

Suy ra $v_2\left(\binom{2^n}{2i+1}\right) = 2^n,\forall i = \overline{0, 2^{n-1} - 1}$.

Do đó có thể đặt $\left(\binom{2^n}{2i+1}\right)^2 = 2^{2n} . x_i$, với $2\nmid x_i$ thì $$\sum_{i=0}^{2^{n-1}-1}\binom{2^{n}}{2i+1} = 2^{2n}\sum_{i=0}^{2^{n-1}-1}x_i^2$$

Mà $\sum x_i^2\vdots 2$ nên ta có điều phải chứng minh.

 




#736359 Chứng minh rằng: Đường tròn đường kính $ST$ trực giao với đường trò...

Đã gửi bởi Hoang72 on 19-12-2022 - 22:03 trong Hình học

Gọi $D$ là tiếp điểm của $(I)$ trên $BC$. $M$ là trung điểm $BC$.

$BI,CI$ cắt $EF$ tại $X,Y$. Theo tính chất quen thuộc, ta thấy $X,Y\in (M;MB)$.

Do đó $(DMXY)$ là đường tròn Euler của $\Delta IBC$.

Tiếp tuyến tại $X,Y$ của $(M)$ cắt nhau tại $H$. Bằng biến đổi góc, ta chứng minh được $HX\parallel TF; HY\parallel TE$.

$DI$ cắt $EF$ tại $J$.

Có $\angle IDX = \angle ICX = \angle IBY - \angle IDY\Rightarrow DI$ là phân giác $\angle XDY$

$\Rightarrow \frac{JX}{JY} = \frac{DX}{DY} = \frac{AC}{AB}$.

Mà $\frac{JF}{JE} = \frac{\sin \widehat{JIF}}{\sin \widehat{JIE}} = \frac{\sin \widehat{ABC}}{\sin\widehat{ACB}}$

Nên ${JX}{JY} = \frac{JF}{JE}\Rightarrow \frac{JX}{XY} = \frac{JF}{FE}\Rightarrow \frac{JX}{JF} = \frac{XY}{FE} =\frac{HX}{TF}$

$\Rightarrow H,J,T$ thẳng hàng.

Dễ thấy $BY,CX,DJ$ đồng quy nên $(SJ,XY) = (SD,CB) = -1$.

Suy ra $J,S$ liên hợp với nhau qua $(DXY)$. Lại có $S$ nằm trên đường đối cực của $H$ nên $HJ$ là đường đối cực của $S$.

Mà $T\in HJ$ nên $S,T$ liên hợp với nhau qua $(DXY)$, tức ta có điều phải chứng minh.

Hình gửi kèm

  • Untitled.png



#736343 $P(x) \vdots Q(x)$

Đã gửi bởi Hoang72 on 18-12-2022 - 20:59 trong Đa thức

a) Ta chứng minh $k=2$.

Thật vậy, giả sử tồn tại vô hạn đa thức nguyên monic có bậc đôi một phân biệt, đều có hệ số của $x^0, x^1,x^2$ lần lượt là $a_0,a_1,a_2$ đều khác $0$ và mỗi đa thức bậc $m$ thì có $m$ nghiệm nguyên.

Xét đa thức $P(x)$ bậc $n$ bất kì thuộc tập các đa thức trên.

Gọi $-x_1,-x_2,...,-x_n$ là các nghiệm nguyên của $P(x)$. Dễ thấy các nghiệm này khác $0$.

Theo hệ thức Viète ta có: $\begin{cases} x_1x_2...x_n = a_0\text{ (1)} \\ \dfrac{1}{x_1} + ... + \dfrac{1}{x_n} = \dfrac{a_1}{a_0} \\ \displaystyle\sum_{1\leq i < j \leq n} \frac{1}{x_ix_j} = \dfrac{a_2}{a_0}\end{cases}$.

Từ $(1)$ ta thấy khi cho $n$ đủ lớn thì $P(x)$ chứa vô số nghiệm có giá trị tuyệt đối bằng $1$.

Mặt khác ta có $\frac{1}{x_1^2} + ... + \frac{1}{x_n^2} = \left(\dfrac{1}{x_1} + ... + \dfrac{1}{x_n}\right)^2 -2\displaystyle\sum_{1\leq i < j \leq n} \frac{1}{x_ix_j} = \frac{a_1^2-2a_2a_0}{a_0^2}$ là một hằng số không phụ thuộc vào $n$.

Điều này vô lí khi cho $n\to+\infty$.

Do đó $k\leq 2$.

Khi $k=2$, ta thấy các đa thức có dạng $(x^2-1)^n . (x-1)$ luôn có chung hệ số của $x^0$ và $x^1$.

b) Đặt $P(x) = x^n + a_{n-1}x^{n-1} +... + a_0$.

Xét hai trường hợp:

+) Tồn tại vô hạn số nguyên dương $a$ để $P(a)$ là luỹ thừa bậc $n$ của một số nguyên:

Xét $Q_1(x) = (nx + a_{n-1} + 1)^n - n^nP(x)$ là đa thức bậc khác $0$ và có hệ số cao nhất dương nên tồn tại $x_1$ để: $Q_1(x) > 0,\forall x > x_1$.

Tương tự xét $Q_2(x) = (nx + a_{n-1}-1)^n - n^nP(x)$ thì tồn tại $x_2$ để $Q_2(x) < 0,\forall x>x_2$.

Suy ra với mọi $x>\max\{x_1,x_2\}$ thì $(nx + a_{n-1} - 1)^n < n^nP(x) < (nx + a_{n-1}+1)^n$.

Tức theo nguyên lí kẹp phải tồn tại vô hạn $x>\max\{x_1,x_2\}$ để $n^nP(x) = (nx + a_{n-1})^n$

$\Rightarrow n^nP(x) = (nx + a_{n-1})^n,\forall x \Rightarrow P(x) = \left(x + \frac{a_{n-1}}{n}\right)^n$.

+) Tồn tại vô hạn số nguyên âm $a$ để $P(a)$ là luỹ thừa bậc $n$ của một số nguyên: Thế thì xét đa thức $R(x) = (-1)^n . P(-x)$ ta có $R$ monic và tồn tại vô hạn $a$ nguyên dương để $R(a)$ là luỹ thừa bậc $n$ của một số nguyên.

 

 




#736302 Tìm hệ số của $x^{2}$ trong $P_{2}(x)...

Đã gửi bởi Hoang72 on 17-12-2022 - 10:25 trong Đa thức

Ta có $P_1(x) = (x-2)^2 - 2=x^2 - 4x + 2$.

Đặt $a_n,b_n,c_n$ lần lượt là hệ số tự do, hệ số của $x$ và hệ số của $x^2$ trong $P_n(x)$.

Thế thì $a_1 = 2; b_1 = -4; c_1 = 1$.

Đồng thời $a_n = a_{n-1}^2 - 2,\forall n\in\mathbb N^*$.

Từ đây, dễ thấy $a_n = 2,\forall n\in\mathbb N^*$.

Đồng nhất hệ số, có $b_n = 2b_{n-1}a_{n-1},\forall n\in\mathbb N^*\Rightarrow b_n = -4^n,\forall n\in\mathbb N^*$.

Tiếp tục đồng nhất hệ số ta có $c_n = 2a_{n-1}c_{n-1} + b_{n-1}^2,\forall n\in\mathbb N^* \Rightarrow c_n = 4c_{n-1} + {16}^{n-1},\forall n\geq 2$

$\Rightarrow c_n - \frac{16^{n}}{12} = 4\left (c_{n-1} - \frac{16^{n-1}}{12}\right),\forall n\in\mathbb N^*$

$\Rightarrow c_n = \frac{4.16^{n-1}}{3} - \frac{4^{n-1}}{3},\forall n\in\mathbb N^*$.

 

 




#736280 $2f(x+y)+f(2x-2y)=f(2x)+f(2y)+2f(x-y), \forall x,y \in \m...

Đã gửi bởi Hoang72 on 16-12-2022 - 16:07 trong Phương trình hàm

Kí hiệu $P(a,b)$ là phép thế $x=a; y=b$ vào phương trình hàm đã cho.

$P(0;0)\Rightarrow f(0) = 2f(0)\Rightarrow f(0) = 0$.

$\begin{aligned} P(x;-x) & \Rightarrow f(4x) = f(2x) + f(-2x) + 2f(2x),\forall x\in\mathbb R & \\ \Rightarrow f(2x) = 3f(x) + f(-x),\forall x\in\mathbb R\end{aligned}$.

Do đó phương trình hàm đã cho tương đương $$2f(x+y) + 3f(x-y) + f(y-x) = 3f(x) + f(-x) + 3f(y) + f(-y) + 2f(x-y),\forall x,y\in\mathbb R$$

$$\Leftrightarrow 2f(x+y) + f(x-y) + f(y-x) = 3f(x) + 3f(y) + f(-x)+f(-y),\forall x,y\in\mathbb R$$

Ở phương trình hàm trên, thay $x$ bởi $-x$ và $y$ bởi $-y$ rồi trừ đi ta được: $f(x+y) - f(-x - y) = f(x) - f(-x) + f(y) - f(-y),\forall x,y\in\mathbb R$.

Suy ra $f(x) - f(-x)$ là hàm cộng tính. Mà $f(x) - f(-x)$ liên tục nên $f(x) - f(-x) = ax,\forall x\in\mathbb R$.

Đặt $g(x) = f(x) - \frac{ax}{2},\forall x\in\mathbb R$ thì $g(x) = g(-x),\forall x\in\mathbb R$, hay $g$ là hàm chẵn.

Đồng thời, ta vẫn có $2g(x+y) + g(2x - 2y) = g(2x) + g(2y) + 2g(x-y),\forall x,y\in\mathbb R$. $(2)$

Vì $g$ có tính chất tương tự hàm $f$ nên $g(2x) = 3g(x)+g(-x) = 4g(x),\forall x\in\mathbb R$.

Kết hợp với $(2)$, ta có $g(x+y) + g(x-y) = 2g(x) + 2g(y)$.

Thay $x = 2y$ ở trên ta có $g(3y) + g(y) = 2g(2y) + 2g(y) = 10g(y),\forall y\in\mathbb R\Rightarrow f(3y) = 9g(y),\forall y\in\mathbb R$.

Bằng quy nạp, dễ dàng chứng minh được $g(nx) = n^2g(x),\forall x\in\mathbb R,n\in\mathbb N^*$.

Từ đó với mọi $a,b\in\mathbb N^*$ thì $g\left(\frac{a}{b}\right) = \frac{g(a)}{b^2} = \frac{a^2g(1)}{b^2}\Rightarrow g(x) = x^2g(1),\forall x\in\mathbb Q^+$.

Do $g$ là hàm chẵn nên $g(x) = x^2g(1),\forall x\in\mathbb Q$.

Mặt khác, $g$ là hàm liên tục, nên $g(x) = x^2g(1),\forall x\in\mathbb R$

$\Rightarrow f(x) = mx^2+nx,\forall x\in\mathbb R$ với $m,n$ là các hằng số thực.

Thử lại ta thấy thoả mãn.




#736099 CM hàm số Dirichlet gián đoạn tại mọi điểm

Đã gửi bởi Hoang72 on 06-12-2022 - 22:16 trong Giải tích

Đặt $f(x) = D(x)$.

Giả sử tồn tại $x_0\in\mathbb R$ mà $f$ liên tục tại $x_0$.

TH1: $x_0\in\mathbb I$: Tồn tại một dãy $(u_n)\subset \mathbb Q$ mà $\lim u_n = x_0$.

Thế thì $ 0 =f(x_0) = \lim_{x\to x_0} f(x) = \lim f(u_n) = 1$. (vô lí)

TH2: $x_0\in\mathbb Q\setminus \{0\}$: Xét dãy số: $a_n = \frac{\sqrt{n^2+1}}{n}x_0,\forall n\in\mathbb N^*$. Thế thì $(a_n)\subset \mathbb I$

$\Rightarrow 1 = f(x_0) = \lim f(a_n) = 0$. (vô lí)

TH3: $x_0 = 0$: Xét dãy số $u_n = \frac{1}{\sqrt{n^2+1}}$. Chứng minh tương tự ta có điều vô lí.

Vậy hàm số Dirichlet gián đoạn tại mọi điểm.




#736065 CM nếu $f$ liên tục và có giới hạn hữu hạn thì bị chặn trên

Đã gửi bởi Hoang72 on 04-12-2022 - 20:46 trong Giải tích

Vậy còn chứng minh $f$ bị chặn dưới trên $\left \lfloor a,+\infty \right )$ thì sao ạ

 

Ta có $f$ đạt GTNN trên $\left [ a,x_{o} \right ]$ tại $m$ nên f bị chặn dưới bởi $m$ trên $\left [a,+\infty \right )$

E trình bày như vậy có đúng không ạ?

Chặn dưới thì bạn chọn GTNN và ta cũng có $f(x) > l - \delta,\forall x > x_0$.




#736062 CM nếu $f$ liên tục và có giới hạn hữu hạn thì bị chặn trên

Đã gửi bởi Hoang72 on 04-12-2022 - 18:27 trong Giải tích

Do $\lim_{x\to+\infty} = l$ nên $\forall \delta > 0,\exists x_0 \geq a: |f(x) - l| < \delta,\forall x > x_0$

$\Rightarrow f(x) \leq \delta + l,\forall x > x_0$.

Đồng thời $f$ liên tục trên $[a; x_0]$ nên nó có giá trị lớn nhất là $M$ trên đoạn này.

Vậy $f$ bị chặn trên bởi $\max\{M, \delta + l\}$.




#736005 Cho tgABC đường cao BE,CF. I là tđ EF, AK vgóc EF. M,N là tđ BE,CF. CM: M,N,I...

Đã gửi bởi Hoang72 on 02-12-2022 - 17:22 trong Hình học

Bạn xem bài toán tổng quát ở đây




#735989 CMR luôn tìm được 5 hàng và 5 cột chứa tất cả 15 quân cờ

Đã gửi bởi Hoang72 on 01-12-2022 - 23:32 trong Tổ hợp và rời rạc

Chọn ra $5$ hàng sao cho $5$ hàng đó chứa số quân cờ là nhiều nhất.

Gọi $a$ là số quân cờ nằm trên $5$ hàng này.

Ta chứng minh $a\geq 10$. Thật vậy, giả sử phản chứng $a< 10$. Thế thì $5$ hàng còn lại chứa ít nhất $6$ quân cờ, do đó tồn tại $1$ hàng trong số này chứa nhiều hơn $1$ quân cờ. Đồng thời, vì $5$ hàng ta chọn lúc đầu chứa ít hơn $10$ quân cờ nên tồn tại $1$ hàng sẽ chứa không quá $1$ quân cờ. Điều này hiển nhiên là mâu thuẫn với cách chọn các hàng sao cho chứa số quân cờ nhiều nhất.

Do đó $5$ hàng này chứa ít nhất $10$ quân cờ. Chọn $5$ cột chứa $5$ quân cờ còn lại ta có đpcm.




#735943 $\sum^n_{k=0}{kP_n(k)}=n!$

Đã gửi bởi Hoang72 on 28-11-2022 - 23:13 trong Toán rời rạc

Ta xét bài toán phụ: Cho $A = \{a_1,a_2,...,a_n\}$ và $B = \{b_1,b_2,...,b_n\}$. Đếm số song ánh $f: A\to B$ sao cho $f(a_i)\neq b_i,\forall i = \overline{1,n}$.

Bằng nguyên lí bao hàm loại trừ, dễ dàng chứng minh được kết quả của bài toán trên là: $\sum_{i=0}^n (-1)^i\frac{n!}{i!}$.

Như vậy, với mỗi $k=\overline{0,n}$ thì: $P_{n}(k) = \frac{n!}{k!}\sum_{i=0}^{n-k}\frac{(-1)^i}{i!}$.

Từ đây, dẫn đến: $\sum_{k=0}^n kP_n(k) = n!\sum_{k=1}^n\sum_{i=0}^{n-k}\frac{(-1)^i}{i!(k-1)!}$.

Ta chỉ cần chứng minh $\sum_{k=1}^n\sum_{i=0}^{n-k}\frac{(-1)^i}{i!(k-1)!} = 1$.

Rõ ràng tổng ở vế trái cũng chính là: $\sum_{k+i< n}\frac{(-1)^i}{i!k!}$.

Ta chứng minh $S_n = \sum_{k+i< n}\frac{(-1)^i}{i!k!}=1$ bằng phép quy nạp.

Khi $n=1$ thì đẳng thức đúng. Giả sử đẳng thức đúng với $n\in\mathbb N^*$.

Ta chứng minh đẳng thức đúng với $n+1$.

Dễ thấy: $S_{n+1} - S_n =\sum_{i+k = n}\frac{(-1)^i}{i!k!} = \frac{1}{n!}\sum_{i=0}^n(-1)^i\binom{n}{i}$.

Ta dễ dàng chứng minh được $\sum_{i=0}^n(-1)^i\binom{n}{i}=0$ bằng cách chứng minh số tập con có chẵn phần tử của $\{1;2;...;n\}$ là $2^{n-1}$.

Suy ra $S_{n+1} - S_n = 0$ hay $S_{n+1}=1$.

Theo nguyên lí quy nạp ta có $S_n=1,\forall n\in\mathbb N^*$.

Vậy $\sum_{k=0}^n kP_n(k) = n!$.